Probability of three events occurring

Click For Summary
A survey at A University revealed that 55% of students use Twitter, 53% use Facebook, and 20% use both Twitter and Facebook. The discussion focuses on calculating the probabilities of various combinations of social media usage, including the percentage of students using both Twitter and Instagram, all three platforms, and those not using any. Confusion arises regarding the interpretation of overlapping probabilities, particularly concerning negative values which are not valid in probability theory. Participants emphasize the importance of correctly applying probability rules and suggest using Venn diagrams for clarity. The conversation highlights the complexities of calculating probabilities in overlapping sets and the necessity for accurate mathematical expressions.
fiksx
Messages
77
Reaction score
1
Homework Statement
probability three event
Relevant Equations
i could find ##P(T \cup F) = P(T)+P(F)-P(T \cap F)=55\% +53\%-20\%=88% ## is this right?

##P(T) =35## but i dont know ##P(T \cap I)##
A questionnaire survey on the use of SNS was conducted for students at A University. As a result,
we got the following:

##55\%## using Twitter ,

##53\%## using Facebook ,

##20\%## using Twitter and facebook both,

##19\%## use both Facebook and
Instagram.

##76\%## use at least Twitter
and / or Instagram

##72\%## use at least one of Facebook
and Instagram

##49\%## use only one of twitter ,facebook, or instagram
At this time, find the next
ratio respectively.

##1##. Percentage of using both
Twitter and Instagram

##2##. Percentage of using all
of Twitter, Facebook and Instagram

##3##. Percentage of not using
either Twitter, Facebook or Instagram
I was confused, for ##P(T\cap F)=20\%## does this also include ##P(T\cap F \cap I )## ?
for ##P(F) ## pnly ##P(F\cap T)##, is it correct only facebook is ##14\%##?
##P(F)=53\% - P(F\cap T) - P(F\cap I)=53\%-19\%-20\% =14\%##?i could find ##P(T \cup F) = P(T)+P(F)-P(T \cap F)=55\% +53\%-20\%=88% ##
is this right?
##P(T) =35##
but i don't know
##P(T \cap I)##

P(F)=14

i manage to find :
##P(T \cap I) = 17## and ##P(I)=38##
but when i count
##P(T \cup F \cup I) = P(T)+P(F)+P(I)-P(T \cap F) - P(T \cap I) - P(I \cap F) + P(T \cap F \cap I) = ##
##49=55+53+38-20-19-17 +P(T \cap F \cap I) ##
##P(T \cap F \cap I)=-41##
 
Physics news on Phys.org
Several important remarks to be made here:

(1) A probability is a number in ##[0,1]##. E.g., either write ##P(A) = 0.1## or ##P(A) = 10 \%##, but not ##P(A) = 10##. The latter is just plain wrong.

(2) Because of ##(1)##, negative probabilities aren't allowed. Your answer ##P(T \cap F \cap I) = -41## doesn't make sense because of two reasons!

The trick to solve this exercise is to write what you are looking for in terms of what you have. For example, you have to write the set ##T \cap I## in terms of what has been given. Drawing Venn diagrams might definitely help.
 
Question: A clock's minute hand has length 4 and its hour hand has length 3. What is the distance between the tips at the moment when it is increasing most rapidly?(Putnam Exam Question) Answer: Making assumption that both the hands moves at constant angular velocities, the answer is ## \sqrt{7} .## But don't you think this assumption is somewhat doubtful and wrong?

Similar threads

  • · Replies 1 ·
Replies
1
Views
1K
Replies
2
Views
1K
  • · Replies 6 ·
Replies
6
Views
2K
  • · Replies 7 ·
Replies
7
Views
2K
  • · Replies 2 ·
Replies
2
Views
2K
Replies
2
Views
2K
Replies
1
Views
2K
  • · Replies 4 ·
Replies
4
Views
3K
Replies
1
Views
2K
  • · Replies 6 ·
Replies
6
Views
3K